You are on page 1of 11

Real Analysis Chapter 6 Solutions Jonathan Conder

3. Since Lp and Lr are subspaces of CX , their intersection is a vector space. It is clear that k · k is a norm (this
follows directly from the fact that k · kp and k · kr are norms). Let hfn i1 p r
n=1 be a Cauchy sequence in L \ L . Since
kfm fn kp  kfm fn k and kfm fn kr  kfm fn k for all m, n 2 N, it is clear that hfn i1 n=1 is a Cauchy sequence
in both Lp and Lr . Let gp 2 Lp and gr 2 Lr be the respective limits of this sequence. Given " 2 (0, 1), there exists
N 2 N such that kfn gp kp < "(p+1)/p for all n 2 N with n N. If n 2 N and n N
Z Z Z
µ(E)"p = "p  |fn gp |p  |fn gp |p < "(p+1)p/p = "p+1 ,
E E

so µ(E) < ", where E := {x 2 X | "p  |fn (x) gp (x)|p } = {x 2 X | "  |fn (x) gp (x)|}. This shows that hfn i1 n=1
converges in measure to gp . If r < 1, a similar argument shows that hfn i1 n=1 converges in measure to gr . Otherwise,
some subsequence of hfn i1n=1 converges to gp almost everywhere, and this subsequence also converges to gr almost
everywhere (since hfn i1 p
n=1 converges uniformly to gr o↵ a set of measure 0). In either case gp = gr , so gp 2 L \ L .
r

Since hfn i1 p r 1 p r
n=1 converges to gp in L and L , it easily follows that hfn in=1 converges to gp in L \ L . This shows that
Lp \ Lr is a Banach space.
Now let q 2 (p, r). There exists 2 (0, 1) such that kf kq  kf kp kf k1r for all f 2 Lp \Lr . In particular, if f 2 Lp \Lr
and kf k = 1, then kf kp  1 and kf kr  1 and hence kf kq  1 11 = 1. This shows that the linear map Lp \Lr ,! Lq
is continuous.

4. Since Lp and Lr are subspaces of CX , their sum is a vector space. Clearly kf k 0 for all f 2 Lp + Lr . Let f 2 Lp + Lr
and a 2 C. If a = 0 then kaf k = k0k = 0 = |a|kf k because

0 = k0kp + k0kr 2 {kgkp + khkr | g 2 Lp , h 2 Lr and f = g + h}.

Otherwise kaf k  kagkp +kahkr = |a|(kgkp +khkr ) for all g 2 Lp and h 2 Lr with f = g+h, and hence kaf k/|a|  kf k.
This implies that |a|kf k = ka 1 af k/|a 1 |  kaf k, which shows that kaf k = |a|kf k. Now let f1 , f2 2 Lp + Lr and
" 2 (0, 1). There exist g1 , g2 2 Lp and h1 , h2 2 Lr such that f1 = g1 + h1 , f2 = g2 + h2 , kg1 kp + kh1 kr < kf1 k + " and
kg2 kp + kh2 kr < kf2 k + ". It follows that

kf1 + f2 k  kg1 + g2 kp + kh1 + h2 kr  kg1 kp + kg2 kp + kh1 kr + kh2 kr < kf1 k + kf2 k + 2",

which shows that kf1 + f2 k  kf1 k + kf2 k. Finally, let f 2 Lp + Lr , and suppose that kf k = 0. Given " 2 (0, 1),
there exist g 2 Lp and h 2 Lr such that f = g + h and kgkp + khkr < "(p+1)/p . It follows that µ(G) < ", where
G := {x 2 X | "  |g(x)|}, because
Z Z Z
µ(G)"p = "p  |g|p  |g|p < "(p+1)p/p = "p+1 .
G G

Similarly µ(H) < " where H := {x 2 X | "  |h(x)|} (if r = 1 then µ(H) = 0 because "(p+1)/p < "). Define
F := {x 2 X | 2"  |f (x)|}, so that F ✓ G [ H. Then µ(F )  µ(G [ H) < 2". It follows that

{x 2 X | 0 < |f (x)|} = [1 1
n=1 \k=n {x 2 X | 2k
1
 |f (x)|},

has measure 0, so f = 0 almost everywhere. This shows that k · k is a norm.


P
Let 1 n=1 fn be an absolutelty convergent series in L + L . For each n 2 N there exist gn 2 L and hn 2 L such
p r p r
P P1
that fn = gn + hn and kgn kp + khn kr < kfn k + 2 n . It follows that 1 g
n=1 n and n=1 hn are absolutely convergent
series in Lp and Lr respectively, so they have limits g 2 Lp and h 2 Lr respectively. This convergence also holds in

1
Real Analysis Chapter 6 Solutions Jonathan Conder

PN PN PN PN
Lp+ Lr ,
because k n=1 gn gk  k n=1 gn gkp + k0kr and k n=1 hn hk  k0kp + k n=1 hn hkr for all
P P1
N 2 N. Therefore 1 f
n=1 n = p r p r
n=1 (gn + hn ) has a limit in L + L , namely g + h. This shows that L + L is a
Banach space.
Now let q 2 (p, r) and let f 2 Lq with kf kq = 1. Define E := {x 2 X | 1 < |f (x)|}, g := f E and h := f Ec . Then
|g|p  |g|q  |f |q and |h|r  |h|q  |f |q , so g 2 Lp and h 2 Lr , which implies that
✓Z ◆1/p ✓Z ◆1/r
kf k = kg + hk  kgkp + khkr  |f |q + |f |q = 11/p + 11/p = 2.

This shows that the linear map Lq ,! Lp + Lr is continuous.

6. Most of the hard work has been done as part of exercise 2.64.

(a) If p1 < 1, define f (x) := x 1/p1 (x) + x 1/p0 1/p1 by | log(x)|.


(0, 12 ) (2,1) (x). Otherwise replace x
(b) If p1 < 1, set f (x) := x 1/p1 | log(x)| 2/p1 +x 1/p0 | log(x)| 2/p0
(0, 12 ) (2,1) . Otherwise, omit the first term.
(c) Set p1 := p0 in part (b).

9. In exercise 3 we showed that convergence in Lp implies convergence in measure (which in turn implies Cauchyness in
measure, hence subsequential convergence almost everywhere). Conversely, let hfn i1 n=1 be a sequence of measurable
functions in CX which converges to f : X ! C in measure. Suppose there exists g 2 Lp such that |fn |  g for
all n 2 N. If hfnk i1 1
k=1 is a subsequence of hfn in=1 , it converges to f in measure, so it has a further subsequence
hfnkj i1 p 1 p
j=1 which converges to f almost everywhere. It follows that h|fnkj | ij=1 converges to |f | almost everywhere,
so by the dominated convergence theorem |f |p 2 L1 , whence f 2 Lp (note that |fn |p  g p for all n 2 N, and g p 2 L1 ).
Furthermore h|fnkj f |p i1 j=1 converges to 0 almost everywhere, and since |fnkj f |p  2p (g p + |f |p ) for all j 2 N,
the dominated convergence theorem implies that hkfnkj f kpp i1 j=1 converges to 0, in which case hkfnkj f kp i 1
j=1 also
converges to 0. This shows that every subsequence of hkfn f kp i1 n=1 has a subsequence which converges to 0, so
hkfn f kp i1n=1 itself converges to 0.

10. Let hfn i1 p p


n=1 be a sequence in L which converges to f 2 L almost everywhere. Suppose that hkfn kp in=1 converges
1

to kf kp . Since |fn f |p  2p (|fn |p + |f |p ), h|fn f |p i1 p p p 1


n=1 and h2 (|fn | + |f | )in=1 converge to 0 and 2
p+1 |f |p almost

everywhere, and
Z ⇣ ⌘p Z
lim 2p (|fn |p + |f |p ) = 2p lim kfn kpp + 2p kf kpp = 2p lim kfn kp + 2p kf kpp = 2p kf kpp + 2p kf kpp = 2p+1 |f |p ,
n!1 n!1 n!1

the generalised dominated convergence theorem implies that hkfn f kpp i1 1


n=1 converges to 0, in which case hkfn f kp in=1
also converges to 0. The converse follows from the reverse triangle inequality.
Pr
13. Let f 2 Lp (Rn , m) and " 2 (0, 1). By Proposition 6.7 there is a simple function g = j=1 aj Ej such that
m([rj=1 Ej ) < 1 and kf gkp < 4" . Fix j 2 {1, . . . , r} and assume without loss of generality that m(Ej ) > 0.
"
There exists bj 2 Q(i) such that kaj Ej bj Ej kp = |aj bj |m(Ej )1/p < 4r . Moreover, by Theorem 2.40(c) there is a
"
finite collection of rectangles whose sides are intervals such that kbj Ej bj Fj kp = |bj |m(Ej 4Fj )1/p < 4r , where Fj
is the union of these rectangles. We may shrink Fj to give a finite collection of rectangles with sides that are intervals
"
with rational endpoints, whose union Gj satisfies kbj Fj bj Gj kp < 4r . It follows that

r
X r
X r
X r
X
f bj Gj  kf gk + kai Ei bi Ei kp + kbi Ei bi Fi kp + kbi Fi bi G i kp < ",
j=1 j=1 j=1 j=1
p

2
Real Analysis Chapter 6 Solutions Jonathan Conder

so the collection of rational linear combinations of characteristic functions of finite unions of rectangles with sides
that are intervals with rational endpoints (which is countable) is dense in Lp (Rn , m).
For each r 2 (0, 1) set fr := Br (0) , and note that kfr fs k1 = 1 for all r, s 2 (0, 1) such that r 6= s (find some
open rectangle inside Br (0)4Bs (0)). If D ✓ L1 (Rn , m) is dense then B1/2 (fr ) \ D 6= ? for all r 2 (0, 1), so there
is an injective function (0, 1) ! D. In particular D is uncountable.

15. Let hfn i1


n=1 be a Cauchy sequence in L . Let ", 2 (0, 1). There exists N 2 N such that kfm
p fn kp < " 1/p for all
m, n 2 N with m n N. If m, n 2 N and m n N then
Z Z Z
µ(E)"p = "p  |fm fn |p  |fm fn |p = kfm fn kpp < (" )(p+1)p/p = "p ,
E E

so µ(E) < , where E := {x 2 X | "  |fm (x) fn (x)|} = {x 2 X | "p  |fm (x) fn (x)|p }. This shows that hfn i1 n=1
is Cauchy in measure. Now let " 2 (0, 1). There exists N 2 N such that kfm fn kp < 2 (p+1)/p "1/p for all m, n 2 N
with m n N. Since {|fn |p }N 1
n=1 is a finite subset of L , it is uniformly integrable by exercise 11 in section 3.2.
R
Hence, there exists 2 (0, 1) such that | E |fn |p | < 2 (p+1) " < " for all n 2 N and E 2 M with n  N and µ(E) < .
If n 2 N and E 2 M such that n > N and µ(E) < , then
Z Z Z Z
|fn |p = |fn |p  2p (|fn fN |p + |fN |p )  2p kfn fN kpp + 2p |fN |p < 2 1 " + 2 1 " = ".
E E E E

This shows that {|fn |p }1


n=1 is uniformly integrable. Now fix n 2 N, set F := {x 2 X | 0 < |fn (x)| } and define
p

Fm := {x 2 X | m 1  |fn (x)|p } for each m 2 N. Then hFm i1 1


m=1 is increasing and [m=1 Fm = F, so
Z ✓Z Z ◆ Z Z Z Z
lim |fn |p = lim |fn |p |fn |p = |fn |p lim |fn |p = |fn |p |fn |p = 0.
m!1 F c m!1 Fm m!1 F F
m m

R
Hence, there exists m 2 N such that c
Fm |fn |p < 2 (p+1) ". Define En := Fm , and note that µ(En ) < 1 because
Z Z Z
1 1
µ(En )m = m  |fn |p  |fn |p < 1.
En En
R R
Therefore µ(E) < 1, where E = [n=1 En . Clearly E c |fn |  E c |fn |p < 2 (p+1) " < " for all n 2 N with n  N. If
: N p
n
n 2 N and n > N then
Z Z Z
|fn |p  2p (|fn fN |p + |fN |p )  2p kfn fN kpp + 2p |fN |p < 2 1 " + 2 1 " = ".
Ec Ec Ec

This shows that hfn i1


n=1 has property (iii).

Conversely, let hfn i1 p


n=1 be a sequence in L which satisfies properties (i)-(iii). Let " 2 (0, 1) and choose E 2 M such
R
that µ(E) < 1 and E c |fn |p < 2 (p+3) "p for all n 2 N. If µ(E) = 0 then
Z Z Z Z Z
p p p p p p p p p p+1 (p+3) p "p
|fm fn | = |fm fn |  2 (|fm | + |fn | ) = 2 |fm | + 2 |fn | < 2 " = < "p
Ec Ec Ec Ec 4
R R
and hence kfm fn kp < " for all m, n 2 N. Otherwise, take 2 (0, 1) such that F |fn | = | F |fn |p | < 2 (p+3) "p for
p

all n 2 N and F 2 M with µ(F ) < . For each m, n 2 N define

1/p
Amn := {x 2 X | "(2µ(E))  |fm (x) fn (x)|},

3
Real Analysis Chapter 6 Solutions Jonathan Conder

so that Z Z Z
p p 1 p 1 "p
|fm fn |  " (2µ(E))  " (2µ(E)) = .
E\Amn E\Amn E 2
There exists N 2 N such that µ(Amn ) < for all m, n 2 N with m n N, in which case
Z Z Z Z
"p
|fm fn |p  2p (|fm |p + |fn |p )  2p |fm |p + 2p |fn |p < 2p+1 (p+3) p
" = .
Amn Amn Amn Amn 4

It follows that
Z Z Z Z
p p p p "p "p "p
|fm fn |  |fm fn | + |fm fn | + |fm fn | < + + = "p ,
Ec E\Amn Amn 4 2 4

and hence kfm fn kp < ", for all m, n 2 N with m n N. This shows that hfn i1 p
n=1 is Cauchy in L .

18. (a) Let f 2 L2 ( ) and suppose that kf k2 = 1. For every simple function : X ! [0, 1], hence every 2 L+ and in
R R R
fact every 2 L1 ( ), it is clear that d = dµ + d⌫. Therefore
Z Z Z Z Z p
f d⌫  |f | d⌫  |f | dµ + |f | d⌫ = |f | d  kf k2 k1k2 = (X)

R
by Hölder’s inequality. This shows that the linear functional f 7! f d⌫ on L2 ( ) is bounded, so there exists
R R R R R R
g 2 L ( ) such that f d⌫ = f g d = f g dµ + f g d⌫, and hence f (1 g) d⌫ = f g dµ, for all f 2 L2 ( ).
2
R R
In fact E f (1 g) d⌫ = E f g dµ for all f 2 L2 ( ) and E 2 M (as f E 2 L2 ( )).
R R R R R R
(b) Note that E g d = E g d = E d⌫ = ⌫(E) 0 and E (1 g) d = E d E g d = (E) ⌫(E) 0
R
for all E 2 M. In particular E Im(g) d = 0 for all E 2 M, which implies that Im(g) = 0 -almost everywhere.
Moreover, ({x 2 X | g(x)  n 1 }) = 0 for all n 2 N, which implies that g 0 -almost everywhere. Similarly
1 g 0, whence 0  g  1, -almost everywhere. In particular 0  g  1 µ-almost everywhere and ⌫-almost
everywhere.
(c) Clearly ⌫s (E) = ⌫(?) = 0 for all E 2 M with E ✓ B c . Moreover
Z Z Z Z
µ(E) = E dµ = E · g dµ = E (1 g) d⌫ = 0 d⌫ = 0

for all E 2 M with E ✓ B. Therefore ⌫s ? µ. Since (1 g) 1 A 0 ⌫-almost everywhere, there is an increasing


sequence of simple functions h n i1
n=1 which converges to (1 g)
1
A ⌫-almost everywhere. Since (1 g)
1
A <1
pointwise, n 2 L2 ( ) for all n 2 N. Therefore
Z Z Z Z
A
⌫a (E) = ⌫(A \ E) = (1 g) d⌫ = lim n (1 g) d⌫ = lim n g dµ = g(1 g) 1 A dµ
E 1 g n!1 E n!1 E E

for all E 2 M, by the monotone convergence theorem. This shows that ⌫a ⌧ µ.

19. Note that n = n 1 {1,...,n} and hence k n k = kn 1 {1,...,n} k1 = 1, by Proposition 6.13. It follows by Alaoglu’s
theorem and Theorem 4.29 that ( n )1 n=1 has a weak* cluster point 2 (l1 )⇤ . Suppose that = g for some g 2 l1 .
1
If k 2 N and " 2 (0, 1), then ( {k} ) = g(k) and hence ( n )1 n=1 is frequently in d {k} (B"/2 (g(k))). In particular,
there exists n 2 N with n1 < 2" and n k such that |g(k) n1 | = |g(k) "
n ( {k} )| < 2 . It follows that g(k) < ", so
1
g = 0. Therefore ( n )1
n=1 is frequently in cN (B1 (0)), which is impossible because n ( N ) = 1 for all n 2 N. This
shows that 6= g for all g 2 l1 .

4
Real Analysis Chapter 6 Solutions Jonathan Conder

R
20. (a) Let g 2 (Lp )⇤ .
There exists g 2 Lq ,
where q = p/(p 1), such that g(h) = hg for all h 2 Lp . Hence, it
:
R R
suffices to show that h fn gi1 n=1 converges to f g. To this end, let " 2 (0, 1). Set M := supn2N kfn kp + 1. Since
R
E 7! E |g|q is a finite measure on M which is absolutely continuous with respect to µ, there exists 2 (0, 1)
R R
such that E |g| = | E |g|q | < 12 ("/4M )q for all E 2 M with µ(E) < . Now define F := {x 2 X | 0 < |g(x)|q }
q

and set Fm := {x 2 X | m 1  |g(x)|q } for each m 2 N. Then hFm i1 1


m=1 is increasing and [m=1 Fm = F, so
Z ✓Z Z ◆ Z Z Z Z
lim |g|q = lim |g|q |g|q = |g|q lim |g|q = |g|q |g|q = 0.
m!1 F c m!1 Fm m!1 F F
m m

R
Hence, there exists m 2 N such that |g|q < 12 ("/4M )q , where A := Fm . Note that µ(A) < 1 because
Ac
Z Z Z
µ(A)m 1 = m 1 |g|q  |g|q < 1.
A A

By Egoro↵’s theorem, it follows that there exists B 2 M such that B ✓ A, µ(B) < and hfn |A\B i1 n=1 converges
R R R
uniformly to f |A\B . In particular (A\B)c |g|q = Ac |g|q + B |g|q < ("/4M )q . By Fatou’s lemma
Z Z
|f |p  lim inf |fn |p = lim inf kfn kpp  M p ,
n!1 n!1

so f 2 Lp and kf kp  M. It follows from Hölder’s inequality that

Z Z !1/q
" "
|(fn f )g|  kfn f kp |g|q  (kfn kp + kf kp ) 
(A\B)c (A\B)c 4M 2

for all n 2 N. Set L := µ(A)kgkpq + 1 and choose N 2 N such that |fn (x) f (x)| < "p /2p L for all x 2 A \ B and
n 2 N with n N. If n 2 N and n N, then
Z Z !1/p ✓ ◆1/p
"p "
|(fn f )g|  |fn f| kgkq  µ(A \ B) p kgkq <
A\B A\B 2 L 2

and hence Z Z Z Z Z
fn g fg  |(fn f )g| = |(fn f )g| + |(fn f )g| < ".
A\B (A\B)c
R R
This shows that h fn gi1
n=1 converges to f g, as required.
(b) For each n 2 N define fn := [n,n+1] . Then supn2N kfn k1 = supn2N 1 = 1 and hfn i1 n=1 converges to 0 pointwise.
R
Define g := E , where E := [1 n=1 [2n, 2n + 1]. Clearly |g|  1 and f !
7 f g dm is in L1 (R, m)⇤ . However
R
h fn g dmi1 1
n=1 does not converge, and hence hfn in=1 does not converge weakly, because
8
Z Z <0, if n is odd
fn g dm = [n,n+1]\E dm =
:1, if n is even.

Similarly, for each n 2 N define xn : N ! C by xn := {n} . Then supn2N kxn k1 = supn2N 1 = 1 and hxn i1 n=1
P1
converges to 0 pointwise. Set g := 2N , so that |g|  1 and hence x 7! m=1 x(m)g(m) is in (l1 )⇤ . However
8
1
X <0, if n is odd
xn (m)g(m) = g(n) =
:1, if n is even,
m=1

5
Real Analysis Chapter 6 Solutions Jonathan Conder

P1 1 1
so h m=1 xn (m)g(m)in=1 does not converge, and hence hxn in=1 does not converge weakly.
Now let hfn i1 n=1 be a sequence in L
1 which converges almost everywhere to f : X ! C. Suppose that
R 1
R
supn2N kfn k1 < 1 and µ is -finite. We aim to show that h gfn in=1 converges to gf for all g 2 L1 . To this end,
let g 2 L1 and set M := supn2N kfn k1 . For each n 2 N set En := {x 2 X | fn (x) > M }. Since [1n=1 En has mea-
sure zero, it is easily shown that |f |  M almost everywhere. It follows that |g(fn f )|  |g|(|fn | + |f |)  2|g|M
R
almost everywhere for all n 2 N, so by the dominated convergence theorem h |g(fn f )|i1 n=1 converges to
R
0 = 0. Since Z Z Z
0 gfn gf  |g(fn f )|
R R
for all n 2 N, the squeeze theorem implies that h gfn i1
n=1 converges to gf, as required.

21. Suppose that (fn )1 p


n=1 converges weakly to f in l (A). If a 2 A then {a} 2 l
p/(p 1) defines a functional p ⇤
a 2 l (A) , so
(fn (a))1 1 1
n=1 = ( a (fn ))n=1 converges to a (f ) = f (a). Therefore (fn )n=1 converges pointwise to f. If 2 lp (A)⇤ then
supn2N |fbn ( )| = supn2N | (fn )| < 1, so supn2N kfn kp = supn2N kfbn k < 1 by the uniform boundedness principle.
The converse is a special case of exercise 20(a).

22. (a) If m, n 2 N and m 6= n then


Z 1
hfm , fn i = cos(2⇡mx) cos(2⇡nx) dx
0
Z
1 1
= (cos(2⇡(m n)x) + cos(2⇡(m + n)x)) dx
2 0
✓ ◆
1 sin(2⇡(m n)x) sin(2⇡(m + n)x) 1
= +
2 2⇡(m n) 2⇡(m + n) 0
✓ ◆ ✓ ◆
1 sin(2⇡(m n)) sin(2⇡(m + n)) 1 sin(0) sin(0)
= + +
2 2⇡(m n) 2⇡(m + n) 2 2⇡(m n) 2⇡(m + n)
= 0.

Moreover, if n 2 N then
Z 1 Z 1 1
1 1 1 sin(4⇡nx) 1
hfn , fn i = cos(2⇡nx)2 dx = (cos(0) + cos(4⇡nx)) dx = + = .
0 2 0 2 2 4⇡n 0 2
p
This shows that h 2fn i1 2
n=1 is an orthonormal sequence in L , so it converges weakly to 0 by exercise 63 of the
previous homework. Therefore hfn i1n=1 converges weakly to 0.
Let x 2 (0, 1/4⇡) and N 2 N. Choose m 2 N with N x 1/4  m. Let n 2 N be the smallest number such that
nx 1/4 > m (which exists because R is Archimedean). Then n > N 1 and hence (n 1)x 1/4  m, in
which case 0 < nx m 1/4  x < 1/4⇡. Therefore |2⇡nx 2⇡m ⇡/2| < 1/2. By the mean value theorem
and the fact that | sin |  1, it follows that | cos(2⇡nx) 1| = | cos(2⇡nx) cos(2⇡m + ⇡/2)| < 1/2 and hence
cos(2⇡nx) > 1/2. This implies that hcos(2⇡nx)i1 1
n=1 does not converge to 0, so hfn in=1 does not converge to 0 on
(0, 1/4⇡). Thus, it is not the case that hfn i1
n=1 converges to 0 almost everywhere.
p
Note that {x 2 R | 1/ 2  cos(2⇡x)} = [m2Z [m + 1/8, m + 3/8] = [m2Z [(8m + 1)/8, (8m + 3)/8] and
p
hence, for each n 2 N, {x 2 X | 1/ 2  cos(2⇡nx)} = [nm=0 1
[(8m + 1)/8n, (8m + 3)/8n]. It follows that
p p P 1
m({x 2 X | 1/ 2  |fn (x) 0|}) m({x 2 X | 1/ 2  cos(2⇡nx)}) = nm=0 2/8n = 2/8 for all n 2 N, which
shows that hfn i1
n=1 does not converge to 0 in measure.

6
Real Analysis Chapter 6 Solutions Jonathan Conder

(b) If x 2 (0, 1] there exists N 2 N such that 1/N < x, in which case fn (x) = 0 for all n 2 N with n N. This
shows that hfn i1n=1 converges to 0 on (0, 1], hence almost everywhere. If " 2 (0, 1) then m({x 2 X | " 
|fn (x) 0|})  m({x 2 X | 0 < |fn (x)|}) = m((0, 1/n)) = 1/n for all n 2 N, which shows that hfn i1 n=1 converges
R
to 0 in measure. Given p 2 [1, 1] it is clear that hfn i1 p
n=1 is a sequence in L , and f 7! f is in (Lp )⇤ by Hölder’s
R
inequality and the fact that X 2 L , where q is the conjugate exponent to p. Since fn = 1 6= 0 for all n 2 N,
q

it follows that hfn i1


n=1 does not converge to 0 weakly in L .
p

27. Define a measurable function K : (0, 1)2 ! C by K(x, y) := (x + y) 1 . Note that K( x, y) = 1 K(x, y) for all

, x, y 2 (0, 1). Moreover,


Z 1 Z 1 Z 1 Z 1
1 1 1
Cp = |K(x, 1)|x 1/p dx = 1+1/p + x1/p
dx  1/p
dx + 1+1/p
dx < 1.
0 0 x 0 x 1 x
It follows by Theorem 6.20 that kT f kp  Cp kf kp for all f 2 Lp .

30. (a) If y 2 (0, 1) then


Z Z Z
1/q 1/q 1
K(xy)x dx = K(z)(z/y) y dz = y 1/q 1
K(z)z 1/p 1
dz = y 1/p
(p 1
)

2 2
and hence, using Hölder’s inequality and the fact that K(xy)f (x) = K(xy)1/q x 1/q x1/q K(xy)1/p f (x),
✓Z ◆ p ✓Z ◆p/q Z
2
K(xy)f (x) dx  K(xy)x 1/q dx xp/q K(xy)f (x)p dx
Z
2
= y 1/q (p 1 )p/q xp/q K(xy)f (x)p dx.

Since p + q = pq and hence p q = pq 2q, it follows by Tonelli’s theorem that


Z ✓Z ◆p Z Z
2
K(xy)f (x) dx dy  y 1/q (p 1 )p/q xp/q K(xy)f (x)p dx dy
ZZ
2
= (p 1 )p/q y 1/q xp/q K(xy)f (x)p dy dx
Z Z
2
= (p 1 )p/q xp/q f (x)p K(xy)y 1/q dy dx
Z
2
= (p 1 )p/q xp/q f (x)p x 1/p (p 1 ) dx
Z
2 2 2
= (p 1 )(p+q)/q x(p q )/(pq ) f (x)p dx
Z
= (p 1 )p x(p q)/q f (x)p dx
Z
= (p 1 )p xp 2 f (x)p dx.

Thus, by Hölder’s inequality (and Tonelli again)


ZZ Z ✓Z ◆ ✓Z ◆1/p
1
K(xy)f (x)g(y) dx dy = K(xy)f (x) dx g(y) dy  (p ) xp 2
f (x)p dx kgkq .

(b) From the long calculation in part (a), if f 2 L2 ((0, 1)) then
Z Z Z 2 Z ✓Z ◆2 Z
|T f (y)|2 dy = K(xy)f (x) dx dy  K(xy)|f (x)| dx dy  ( 12 )2 x0 |f (x)|2 dx

7
Real Analysis Chapter 6 Solutions Jonathan Conder

and hence T f 2 L2 ((0, 1)) with kT f k2  ( 12 )kf k2 . Therefore T : L2 ((0, 1)) ! L2 ((0, 1)) is well-defined, and
it is clearly linear. Moreover kT k  ( 12 ).

32. Since |K|2 2 L1 (µ ⇥ ⌫), Fubini’s theorem implies that |Kx |2 2 L1 (⌫) for almost all x 2 X. Therefore Kx 2 L2 (⌫) for
R
almost all x 2 X. By Hölder’s inequality |K(x, y)f (y)| d⌫(y)  kKx k2 kf k2 , and hence T f (x) converges absolutely,
for almost all x 2 X. Fubini’s theorem implies that K y 2 L2 (µ) and hence K y f (y) 2 L2 (µ), for almost all y 2 Y.
Define k : Y ! [0, 1) by k(y) := kK y k2 . Then
Z ZZ ZZ
kkk22 = |k(y)|2 d⌫(y) = |K y (x)|2 dµ(x) d⌫(y) = |K(x, y)|2 dµ(x) d⌫(y) = kKk22

and hence, by Hölder’s inequality


Z Z
kK y f (y)k2 d⌫(y) = kK y k2 |f (y)| d⌫(y) = kkf k1  kkk2 kf k2 = kKk2 kf k2 .

This implies that y 7! kK y f (y)k2 is in L1 (⌫), so by Minkowski’s inequality for integrals


Z Z 2 ✓Z ◆2
|T f (x)|2 dµ(x) = K(·, y)f (y) d⌫(y)  kK(·, y)f (y)k2 d⌫(y)  (kKk2 kf k2 )2 .
2

This shows that T f 2 L2 (µ) and kT f k2  kKk2 kf k2 .

33. Since Lq ((0, x)) ✓ L1 ((0, x)) for all x 2 (0, 1), it is clear that T is a well-defined linear map from Lq ((0, 1)) to C(0,1) .
Rx
If f 2 Lq ((0, 1)), the map x 7! 0 f is absolutely continuous on (0, 1), so T f is continuous. Given " 2 (0, 1) there
R1 R
exists n 2 N such that n |f |q < ( 2" )q , by continuity of the finite measure E 7! E |f |q . Now
Z 1 Z 1
|T f (x) T (f (0,n) )(x)| = x 1/p f x 1/p f (0,n)
Z 01 0

 x 1/p |f f (0,n) |
Z0
= x 1/p (0,x) |f [n,1) |

1/p
x k (0,x) kp kf [n,1) kq
"
<
2
for all x 2 (0, 1), by Hölder’s inequality. In particular, if x 2 (0, 1) and |T f (x)| ", then
Z n
"
 |T (f (0,n) )(x)|  x 1/p |f |
2 0

and hence x  ( 2" kf (0,n) k1 )p . This implies that {x 2 X | "  |T f (x)|} is bounded, thus compact (because |T f | is
continuous), so T f 2 C0 ((0, 1)). Moreover, by Hölder’s inequality
Z Z
|T f (x)| = x 1/p f (0,x)  x 1/p |f (0,x) |  x 1/p k (0,x) kp kf kq = kf kq

for all x 2 (0, 1). This shows that kT f ku  kf kq , so T is a bounded linear map from Lq ((0, 1)) to C0 ((0, 1)).

34. Set q := p/(p 1). Suppose p > 2, so that p = (p 1)q > q and hence q/p < 1. By Hölder’s inequality
Z 1 ✓Z 1 ◆1/p ✓Z 1 ◆1/q
|f 0 |  x|f 0 (x)|p dx x q/p
dx <1
0 0 0

8
Real Analysis Chapter 6 Solutions Jonathan Conder

R1 0
and hence 2f0 Define g : [0, 1] ! C by g(0) = f (1)
L1 ([0, 1]). : :
0 f and g|(0,1) = f. Then g is di↵erentiable almost
everywhere on [0, 1], and g 0 2 L1 ([0, 1]) because g 0 = f 0 almost everywhere. Moreover
Z 1 Z 1 Z 1 Z ✏
g(✏) g(0) = f (✏) f (1) + f0 = f0 + f0 = f0
0 ✏ 0 0

for all ✏ 2 (0, 1). This clearly also holds for ✏ 2 {0, 1}, so g is absolutely continuous on [0, 1] and hence limx!0 f (x) =
limx!0 g(x) = g(0). If p = 2, ✏ 2 (0, 12 ) and 2 (✏, 2✏), then Hölder’s inequality implies that
s s s
Z Z 1 Z 2✏ Z 1 Z 1
|f 0 |  x|f 0 (x)|2 dx · x 1 dx = x|f 0 (x)|2 dx · (log(2✏) log(✏)) = x|f 0 (x)|2 dx · log(2)
✏ 0 ✏ 0 0
qR
1
and hence (where M := 0 x|f 0 (x)|2 dx · log(2))
R
0 ✓ Z ◆
|f (✏)| f( ) ✏ f
0 1/2
= 1/2
 | log(✏)| 1/2
|f ( )| + |f 0 |  | log(✏)| 1/2
(|f ( )| + M )
| log(✏)| | log(✏)| ✏

36. Suppose f 2 weak Lp and µ({x 2 X | 0 6= f (x)}) < 1. Fix q 2 (0, p). Define E0 := {x 2 X | 0 < |f (x)|  1}, and for
each n 2 N set En := {x 2 X | 2n 1 < |f (x)|  2n }. By the monotone convergence theorem
Z Z X 1 X1 1
X 1
X
q n q nq nq n 1
|f |  (2 ) En = 2 µ(En )  µ(E0 ) + 2 f (2 )  µ(E0 ) + 2nq+p np [f ]pp < 1,
n=0 n=0 n=1 n=1
P1
because E0 ✓ {x 2 X | 0 6= f (x)} and ↵p f (↵)  [f ]pp for all ↵ 2 (0, 1) (note that n=1 2
nq np is a convergent
geometric series). This shows that f 2 Lq .
Now suppose that f 2 (weak Lp ) \ L1 , and fix q 2 (p, 1). Define E0 := {x 2 X | 1 < |f (x)|} and for each n 2 N set
En := {x 2 X | 2 n < |f (x)|  21 n }. Then
Z Z 1
!
X
q q
|f |  kf k1 E0 + (21 n )q En
n=1
1
X
= kf kq1 µ(E0 ) + q nq
2 µ(En )
n=1
1
X
 kf kq1 f (1) + 2q nq
f (2
n
)
n=1
1
X
 kf kq1 [f ]pp + q nq+np
2 [f ]pp
n=1

and hence f 2 Lq . It is obvious that f 2 Lq for the case q = 1.


P
38. Let f : X ! C be a measurable function and suppose that n2Z 2np f (2n ) < 1. For each n 2 Z define En := {x 2
X | 2n < |f (x)|  2n+1 }. By the monotone convergence theorem
Z Z X X X
p n+1 p p np p
|f |  (2 ) En = 2 2 µ(En )  2 2np f (2n ) < 1
n2Z n2Z n2Z

and hence f 2 Lp . Conversely, suppose that f 2 Lp . By Proposition 6.24


X Z 1X
2np f (2n ) = 2np n+1 f (2n ) (2n 1 ,2n ]

n2Z 0 n2Z

9
Real Analysis Chapter 6 Solutions Jonathan Conder

Z 1 X
p
2 2(n 1)(p 1)
f (↵) (2n 1 ,2n ] (↵) d↵
0 n2Z
Z 1 X
p
2 ↵p 1
f (↵) (2n 1 ,2n ] (↵) d↵
0 n2Z
Z 1
p
=2 ↵p 1
f (↵) (0,1) (↵) d↵
Z0 1
= 2p ↵p 1
f (↵) d↵
0
= 2p p 1
kf kpp
< 1.

Pn
39. If = i=1 ai Ei is simple with (0) < 1 and ↵ 2 (0, 1), then (↵) = 0 if ↵ max{|a1 |, . . . , |an |} and
(↵) = (0) if ↵ < min{|a1 |, . . . , |an |}. We may assume that ai 6= 0 for all i 2 {1, . . . , n}, so

lim ↵p (↵) = lim 0 = 0 and lim ↵p (↵) = lim ↵p (0) = 0.


↵!1 ↵!1 ↵!0 ↵!0

"
Given " 2 (0, 1), we may choose 2 Lp such that kf kp < 2p+1
. By Chebyshev’s inequality
⇣↵⌘ ⇣↵⌘ ⇣ ↵ ⌘p ⇣↵⌘ " "
0  ↵p f (↵)  ↵
p
f + ↵p  2p kf kpp + 2p < + ="
2 2 2 2 2 2
provided that ↵ 2 (0, 1) is sufficiently small (large). Therefore lim↵!0 ↵p f (↵) = 0 (lim↵!1 ↵p f (↵) = 0).

43. Clearly
Z
1 m((x r, x + r) \ (0, 1))
H (0,1) (x) = sup | (0,1) (y)| dy = sup .
r2(0,1) m((x r, x + r)) (x r,x+r) r2(0,1) 2r

If x 2 ( 1, 0] then 8
>
> 0 r 2 (0, x)
>
<
m((x r, x + r) \ (0, 1)) = x + r r 2 [ x, 1 x]
>
>
>
:1 r 2 (1 x, 1).
x 1 1
Since r 7! 2r + 2 is increasing on [ x, 1 x], it follows that H (0,1) (x) = 2 2x . Given x 2 (0, 12 ]
8
>
> 2r r 2 (0, x)
>
<
m((x r, x + r) \ (0, 1)) = x + r r 2 [x, 1 x]
>
>
>
:1 r 2 (1 x, 1).

Since x + r  2r for all r 2 [x, 1 x] and 1  2r for all r 2 (1 x, 1), it follows that H (0,1) (x) = 1. Similarly
8
>
> 2r r 2 (0, 1 x)
>
<
m((x r, x + r) \ (0, 1)) = 1 x + r r 2 [1 x, x]
>
>
>
:1 r 2 (x, 1)

10
Real Analysis Chapter 6 Solutions Jonathan Conder

whenever x 2 ( 12 , 1), and hence H (0,1) (x) = 1. Given x 2 [1, 1)


8
>
> 0 r 2 (0, x 1)
>
<
m((x r, x + r) \ (0, 1)) = 1 x + r r 2 [x 1, x]
>
>
>
:1 r 2 (x, 1).

1 x 1 1
Since r 7! 2r + 2 is increasing on [x 1, x], it follows that H (0,1) (x) = 2x . Hence, if p 2 [1, 1)
Z Z 0 Z 1 Z 1
p dx dx
|H (0,1) | = + dx + ,
1 (2 2x)p 0 1 (2x)p

which is finite i↵ p 6= 1. Also |H (0,1) |  1 so H (0,1) 2 Lp for all p 2 (1, 1], but not for p = 1. Moreover
H (0,1) 2 weak L1 because x 7! x1 is in weak L1 . Continuing from above
Z 0 1
p 1 1 1 1
|H (0,1) | = +1+ = +1
2(1 p)(2 2x)p 1
1 2(1 p)(2x)p 1
1 2(p 1)2p 1 2(1 p)2p 1

and hence kH (0,1) kp = 1 + (p 1) 1 21 p for all p 2 (1, 1), so kH (0,1) kp behaves like (p 1) 1 as p ! 1. It is
obvious that k (0,1) kp = 1 for all p 2 [1, 1].

11

You might also like